1
$\begingroup$

Let $x_i,y_i\in\mathbb{R}$, $a_i\in[0,1]$, with $y_1>y_2$ and $y_3>y_4$. We also have $(y_1-y_3)^2 + (y_2 -y_4)^2 <\varepsilon $. I need to solve the following constrained optimization problem

$$\min_{x_1,x_2,x_3,x_4}\sum_{i = 1}^4 a_i(x_i-y_i)^2/2$$ $$\text{subject to } x_2\geq x_1,$$ $$ \quad \quad \quad \; x_4 \geq x_3$$ $$ \quad \quad \quad \;\quad \quad \quad \;\quad \quad \quad \; (x_1-x_3)^2 + (x_2-x_4)^2\leq \varepsilon$$

Attempted solution:

$L(\lambda,x) = \sum_{i = 1}^4 a_i(x_i-y_i)^2/2 + \lambda_1(x_1-x_2)+ \lambda_2(x_3-x_4) + \lambda_3( (x_1-x_3)^2 + (x_2-x_4)^2-\varepsilon)$

Differentiating and assuming binding constraints ($x_1 = x_2$, $x_3 = x_4$$\varepsilon = (x_1-x_3)^2 + (x_2-x_4)^2)$ we get

$$x_k = \frac{\sum_i{a_i y_i}\mp \sqrt{\varepsilon/2}}{\sum_i a_i }, \text{ for } k = 1,2$$ and $$x_k = \frac{\sum_i{a_i y_i}\pm \sqrt{\varepsilon/2}}{\sum_i a_i }, \text{ for } k = 3,4$$

Is the procedure correct? Thanks!

$\endgroup$

1 Answer 1

1
$\begingroup$

That is the right approach, but you have a few errors:

  1. The last term of $L(\lambda,x)$ should instead be $\lambda_3((x_1-x_3)^2 + (x_2-x_4)^2 - \varepsilon)$
  2. You have used the index $i$ in two different ways ($x_i$ and $\sum_i$) in the same formula.
  3. Your last formula has $i=1,2$, but you meant $i=3,4$.
$\endgroup$
2
  • $\begingroup$ Corrected, Thanks! Do you think the result is correct? Should I consider the other cases? $\lambda_1 \neq 0$, $\lambda_2 = 0$ and vice-versa? $\endgroup$ Commented May 30, 2021 at 21:18
  • $\begingroup$ I didn't work out the details. Why don't you show the rest of your work? $\endgroup$ Commented May 30, 2021 at 23:33

You must log in to answer this question.

Start asking to get answers

Find the answer to your question by asking.

Ask question

Explore related questions

See similar questions with these tags.